LSAT and Law School Admissions Forum

Get expert LSAT preparation and law school admissions advice from PowerScore Test Preparation.

 Administrator
PowerScore Staff
  • PowerScore Staff
  • Posts: 8916
  • Joined: Feb 02, 2011
|
#40562
Complete Question Explanation
(The complete setup for this game can be found here: lsat/viewtopic.php?t=15423)

The correct answer choice is (B)

The question stem establishes that V performs in slot three. Accordingly, from the first rule, Z must then perform later than V, and thus W and Y must be the other two bands that perform in the grouping of the first three slots (because, as determined during the setup, the first three bands to perform must come from the group of V, Y, and W/Z). The last three bands to perform are then U, X, and Z, with the additional restriction that Z :longline: X from the second rule:
PT70 -Game_#1_#3_diagram 1.png
Answer choice (A): This answer choice is incorrect because X could perform in slot five and U could perform in slot six.

Answer choice (B): This is the correct answer choice.

Answer choice (C): This answer choice is incorrect because U could perform in slots four or five while X performs in slot six.

Answer choice (D): This answer choice is incorrect because W could perform in slot one and Y could perform in slot two.

Answer choice (E): This answer choice is incorrect because U could perform in slot four and Z could perform in slot five.
You do not have the required permissions to view the files attached to this post.

Get the most out of your LSAT Prep Plus subscription.

Analyze and track your performance with our Testing and Analytics Package.